Is there a mistake in the assignment?

Click For Summary
SUMMARY

The discussion revolves around a differential equation with initial conditions that lead to a trivial solution, specifically y(t) = 0. The tutor highlights that the constants c_1 and c_2 cannot be determined from the given conditions, resulting in the equations -c_2 = -3c_1 and c_2 = -3c_1. Further analysis reveals that the boundary conditions resemble Sturm-Liouville type conditions, and the eigenvalue λ = 9 does not yield a non-zero solution, confirming the triviality of the solution.

PREREQUISITES
  • Understanding of differential equations, specifically second-order linear equations.
  • Familiarity with Sturm-Liouville theory and eigenvalue problems.
  • Knowledge of boundary conditions and their implications on solutions.
  • Ability to analyze and interpret mathematical graphs related to eigenvalues.
NEXT STEPS
  • Study Sturm-Liouville theory in detail to understand its applications in differential equations.
  • Learn about eigenvalue problems and how to determine eigenvalues and eigenfunctions.
  • Explore numerical methods for finding solutions to differential equations.
  • Investigate the implications of boundary conditions on the existence of non-trivial solutions.
USEFUL FOR

Mathematics tutors, students studying differential equations, and anyone interested in advanced topics in mathematical analysis and eigenvalue problems.

kostoglotov
Messages
231
Reaction score
6

Homework Statement



I'm actually a tutor, and a student of mine at uni has the following differential equation with initial conditions to solve

ptuymQv.gif


imgur link: http://i.imgur.com/ptuymQv.gif

From y(t) = c_1sin(3t) + c_2cos(3t), it is not possible to solve for the constants using the given initial conditions. I always wind up with -c_2 = -3c_1 and c_2 = -3c_1.

Am I missing something or are those init conditions NP?

Homework Equations

The Attempt at a Solution

 
Physics news on Phys.org
kostoglotov said:

Homework Statement



I'm actually a tutor, and a student of mine at uni has the following differential equation with initial conditions to solve

ptuymQv.gif


imgur link: http://i.imgur.com/ptuymQv.gif

From y(t) = c_1sin(3t) + c_2cos(3t), it is not possible to solve for the constants using the given initial conditions. I always wind up with -c_2 = -3c_1 and c_2 = -3c_1.

Am I missing something or are those init conditions NP?

Homework Equations

The Attempt at a Solution

If you continue with your work, you get ##c_1 = c_2 = 0##. That means that the solution to the DE is ##y(t) = 0\sin(3t) + 0\cos(3t) \equiv 0##.
That might not have been what the author of the problem intended, but it's a valid solution.
 
Last edited:
This response may well be more advanced than what the OP needs, but I'm giving it for others that might read this thread.

Those boundary conditions look like Sturm-Liouville type B.C.'s, and you wouldn't normally expect a non-zero solution for any old value of the constant, which is ##9## in this problem. For example, consider the eigenvalue problem$$
y''+\mu^2 y = 0$$ $$y(0)+y'(0) = 0,~~y(\pi)-y'(\pi)=0$$Sturm-Liouville theory tells us there should be infinitely many eigenvalues ##\lambda_n=\mu_n^2##, with corresponding non-zero eigenfunctions (solutions). Is one of the eigenvalues ##\lambda = 9##? The answer is "no". Without boring you with the details, if you work out the details, the ##\mu_n## are solutions of the equation$$\tan(\pi \mu) = \frac{2\mu}{1-\mu^2}$$They can be found numerically and are where the functions ##\tan(\pi \mu)## and ##\frac{2\mu}{1-\mu^2}## in the picture below cross.
eigenvalues.jpg


Unfortunately, ##9## isn't one of the eigenvalues, which is why there is only the trivial solution.
[Edit, added]: Note ##\lambda = 9## would correspond to ##\mu = 3##.
 
Last edited:
  • Like
Likes pasmith
LCKurtz said:
View attachment 107655

Unfortunately, ##9## isn't one of the eigenvalues, which is why there is only the trivial solution.

Unfortunate that the graph doesn't clearly demonstrate this by having the horizontal axis include \mu = 9.
 
pasmith said:
Unfortunate that the graph doesn't clearly demonstrate this by having the horizontal axis include \mu = 9.

[Edit] This may need correcting, stay tuned...
OK, the below is corrected.
@pasmith: I was apparently asleep when I first replied. For ##\lambda = 9## we need ##\mu## near ##3##, which is illustrated in my previous post. The ##\mu## value nearest ##3## is 2.780188423. This leads to a solution of y = sin(2.780188423*x)-2.780188423*cos(2.780188423*x). Here's a graph:

solution.jpg
 
Last edited:
Question: A clock's minute hand has length 4 and its hour hand has length 3. What is the distance between the tips at the moment when it is increasing most rapidly?(Putnam Exam Question) Answer: Making assumption that both the hands moves at constant angular velocities, the answer is ## \sqrt{7} .## But don't you think this assumption is somewhat doubtful and wrong?

Similar threads

  • · Replies 9 ·
Replies
9
Views
2K
  • · Replies 6 ·
Replies
6
Views
2K
Replies
5
Views
2K
  • · Replies 9 ·
Replies
9
Views
2K
  • · Replies 1 ·
Replies
1
Views
3K
Replies
3
Views
2K
  • · Replies 2 ·
Replies
2
Views
1K
  • · Replies 1 ·
Replies
1
Views
2K
  • · Replies 2 ·
Replies
2
Views
9K
  • · Replies 2 ·
Replies
2
Views
2K